2018 AMC 10B 试题 第1题
问题 计算 \(\frac{{\left( 2112 - 2021 \right)}^{2}}{169}\) 的值。
(A) 7 (B) 21 (C) 49 (D) 64 (E) 91
问题 计算 \(\frac{{\left( 2112 - 2021 \right)}^{2}}{169}\) 的值。
(A) 7 (B) 21 (C) 49 (D) 64 (E) 91
步骤 2:分解分子和分母 观察 \(91\) 和 \(169\) 的关系: \[ 91 = 7 \times 13 \] \[ 169 = 13^2 \] 代入原式: \[ \frac{(7 \times 13)^2}{13^2} \]
步骤 3:化简分式 利用指数运算法则 \((a \cdot b)^2 = a^2 \cdot b^2\) : \[ \frac{7^2 \times 13^2}{13^2} \] 约去分子和分母的公因子 \(13^2\) : \[ 7^2 = 49 \]
门卡拉有一张 \( 4 \times 6 \) 的索引卡。如果她将其中一条边的长度缩短 1 英寸,这张卡的面积将变为 18 平方英寸。那么,如果她改为将另一条边的长度缩短 1 英寸,这张卡的面积将是多少平方英寸?
设原始卡片的长为 \(l\) 、宽为 \(w\) ,且 \(l \times w = 4 \times 6 = 24\) 平方英寸。
因此,第一次缩短的是长度为 4 的边。
面积计算: \[ 4 \times 5 = 20 \]
假设黏土球在装入立方体前可以被重塑但不可压缩,半径为 \(2\) 的黏土球最多能完全装入边长为 \(6\) 的立方体中多少个?
(A) 3 (B) 4 (C) 5 (D) 6 (E) 7
关键点:
代入半径 \(r = 2\) : \[ V_{\text{球}} = \frac{4}{3}\pi \cdot 2^3 = \frac{4}{3}\pi \cdot 8 = \frac{32}{3}\pi \]
代入边长 \(a = 6\) : \[ V_{\text{立方体}} = 6^3 = 216 \]
已知 \(\pi \approx 3.1416\) ,所以: \[ 4\pi \approx 12.5664 \]
计算上下界:
因此: \[ 6.23 < \frac{81}{4\pi} < 6.75 \]
由于球数必须是整数,且 \(\frac{81}{4\pi} < 7\) ,所以最大可能的整数球数为 \(6\) 。
(A) \ \( 2\\frac{3}{4} \\) (B) \ \( 3\\frac{3}{4} \\) (C) \ \( 4\\frac{1}{2} \\) (D) \ \( 5\\frac{1}{2} \\) (E) \ \( 6\\frac{3}{4} \\)
学校区域行驶时间: \[ t_s = \frac{L_s}{v_s} = \frac{\frac{1}{2}}{20} = \frac{1}{40} \ \text{小时} \]
其余路段行驶时间: \[ t_r = \frac{L_r}{v_r} = \frac{\frac{9}{2}}{40} = \frac{9}{80} \ \text{小时} \]
路线 B 总时间: \[ t_B = t_r + t_s = \frac{9}{80} + \frac{1}{40} \] 通分(分母为 80): \[ t_B = \frac{9}{80} + \frac{2}{80} = \frac{11}{80} \ \text{小时} \]
问题 六位数 \(\underline{2}\underline{0}\underline{2}\underline{1}\underline{0}\underline{\mathrm{A}}\) 仅当数字 \(\mathrm{A}\) 取某个特定值时是质数。请问 \(\mathrm{A}\) 的值是多少?
(A) 1 (B) 3 (C) 5 (D) 7 (E) 9
首先,我们列出所有可能的六位数形式: \[ 20210A \] 其中 \(A\) 是 0 到 9 的数字,但题目给出的选项是 1, 3, 5, 7, 9(均为奇数,因为若 \(A\) 为偶数,则整个数能被 2 整除,不是质数)。
我们逐一分析每个选项:
鸸鹋埃尔默在乡村道路上每两个相邻电线杆之间需要走44步等长的步伐。鸵鸟奥斯卡用12次等长的跳跃就能覆盖相同的距离。电线杆间距均匀,沿着这条路第41根电线杆与第一根电线杆正好相距1英里(5280英尺)。奥斯卡的跳跃比埃尔默的步伐长多少英尺?
(A) 8 (B) 9 (C) 10 (D) 11 (E) 15
步骤1:计算相邻电线杆之间的距离 从第1根电线杆到第41根电线杆,共有 \(41 - 1 = 40\) 个间隔。 每个间隔的距离为: \[ \frac{5280}{40} = 132 \text{ 英尺} \]
步骤2:计算埃尔默的步伐长度 埃尔默在两个相邻电线杆之间走44步,因此他的步伐长度为: \[ \frac{132}{44} = 3 \text{ 英尺} \]
步骤3:计算奥斯卡的跳跃长度 奥斯卡在两个相邻电线杆之间用12次跳跃覆盖相同距离,因此他的跳跃长度为: \[ \frac{132}{12} = 11 \text{ 英尺} \]
步骤4:计算跳跃与步伐的长度差 奥斯卡的跳跃比埃尔默的步伐长: \[ 11 - 3 = 8 \text{ 英尺} \]
(A) 170 (B) 174 (C) 其他选项未列出 (D) 170 (E) 174
题目给出正方形 \(ABCD\) ,点 \(E\) 与点 \(A\) 在直线 \(CD\) 所确定的相对半平面上,且 \(\angle CDE = 110^\circ\) 。点 \(F\) 在线段 \(AD\) 上,满足 \(DE = DF\) ,因此 \(\triangle DEF\) 是等腰三角形。要求计算 \(\angle AFE\) 的度数。
\[ \boxed{170} \]
答案(B):设一个两位可爱数为 \( \underline{a}\underline{b} \) ,其值为 \( 10a + b \) 。因此有 \( 10a + b = a + b^{2} \) ,即 \( 9a = b(b - 1) \) 。由此可得 9 整除 \( b(b - 1) \) 。若 \( 3 \mid b \) 且 \( 3 \mid (b - 1) \) ,则 \( 3 \mid 1 \) ,矛盾。因此要么 \( 9 \mid b \) ,要么 \( 9 \mid (b - 1) \) 。此外,由于 \( a \neq 0 \) ,可得 \( b > 1 \) 。因此 \( b \) 唯一可能的取值为 9。此时 \( 9a = 9 \cdot 8 \) ,即 \( a = 8 \) ,故唯一的两位可爱数是 89。
步骤 1:建立方程 由 \(10a + b = a + b^2\) ,移项得: \[ 10a - a = b^2 - b \] \[ 9a = b(b - 1) \]
步骤 2:分析整除性 由于 \(9a = b(b - 1)\) ,且 \(a\) 为正整数,因此 \(9\) 必须整除 \(b(b - 1)\) ,即: \[ 9 \mid b(b - 1) \]
步骤 3:排除矛盾情况 若 \(3 \mid b\) 且 \(3 \mid (b - 1)\) ,则 \(3 \mid 1\) ,矛盾。 因此, \(b\) 和 \(b - 1\) 不能同时被 3 整除,即 \(9\) 必须整除 \(b\) 或 \(b - 1\) 中的一个。
步骤 4:确定 \(b\) 的可能取值
结合 \(a \neq 0\) 及 \(9a = b(b - 1) > 0\) ,得 \(b > 1\) 。 因此, \(b\) 的可能取值为 \(b = 9\) 。
步骤 5:求解 \(a\) 代入 \(b = 9\) 到方程 \(9a = b(b - 1)\) : \[ 9a = 9 \times 8 = 72 \] \[ a = 8 \]
步骤 6:验证 两位数为 \(89\) ,计算: \[ 8 + 9^2 = 8 + 81 = 89 \] 满足条件。
最终答案是 1,对应选项 (B)。
(B) \( \frac{4}{9} \) (C) \( \frac{5}{9} \) (D) \( \frac{9}{16} \) (E) \( \frac{5}{8} \)
设出现奇数的概率为 \( p \) ,则出现偶数的概率为 \( 3p \) 。由于所有可能结果的概率之和为1,我们有: \[ p + 3p = 1 \] 解得: \[ 4p = 1 \quad \Rightarrow \quad p = \frac{1}{4} \] 因此:
(A) 70 (B) 84 (C) 105 (D) 126 (E) 140
\[ 210 \times (1 - v) = L \] \[ \text{(1)} \quad 210(1 - v) = L \]
\[ 42 \times (1 + v) = L \] \[ \text{(2)} \quad 42(1 + v) = L \]
代入 (2): \[ L = 42 \times \left( 1 + \frac{2}{3} \right) = 42 \times \frac{5}{3} = 14 \times 5 = 70 \]
(A) 0 (B) 1 (C) 2 (D) 3 (E) 4
我们需要计算 \( N \mod 5 \) 。由于 \( 9 \equiv -1 \pmod{5} \) ,所以:
分别计算各项的余数:
将所有余数相加: \[ 2 + (-7) + 6 + (-5) + 2 = -2 \equiv 3 \pmod{5} \]
"每个球的颜色都与其余 5 个球中超过一半的球颜色不同"意味着:
考虑所有可能的涂色情况:
分析满足条件的情况:
3黑3白的排列方式数: \(\binom{6}{3} = 20\)
因此,唯一满足条件的情况是 3黑3白。
由于题目只要求推演步骤,且已知答案对应选项,这里给出计算过程。
(A) \(\frac{1}{6}\) (B) \(\frac{1}{4}\) (C) \(\frac{5}{16}\) (D) \(\frac{1}{2}\)
等腰三角形 \(ABC\) 中, \(AB = AC = 3\sqrt{6}\) ,且一个半径为 \(5\sqrt{2}\) 的圆与直线 \(AB\) 相切于点 \(B\) ,与直线 \(AC\) 相切于点 \(C\) 。求经过顶点 \(A\) 、 \(B\) 和 \(C\) 的圆的面积是多少?
(A) \(24\pi\) (B) \(25\pi\) (C) \(26\pi\) (D) \(27\pi\) (E) \(28\pi\)
考虑四边形 \(ABOC\) ,其中 \(\angle ABO\) 和 \(\angle ACO\) 均为直角,因此: \[ \angle ABO + \angle ACO = 180^\circ. \] 这说明四边形 \(ABOC\) 的对角互补,因此四边形 \(ABOC\) 是圆内接四边形。
由于四边形 \(ABOC\) 内接于圆,且 \(\angle ABO = 90^\circ\) ,根据圆内接四边形的性质,对角互补,可得 \(\angle AOC = 90^\circ\) 。进一步,由对称性可知, \(AO\) 是经过点 \(A\) 、 \(B\) 、 \(C\) 的圆的直径。
接下来,计算 \(AO\) 的长度。在直角三角形 \(ABO\) 中,已知: \[ AB = 3\sqrt{6}, \quad OB = 5\sqrt{2}. \] 应用勾股定理: \[ AO = \sqrt{AB^2 + OB^2} = \sqrt{(3\sqrt{6})^2 + (5\sqrt{2})^2}. \] 计算各项: \[ (3\sqrt{6})^2 = 9 \times 6 = 54, \quad (5\sqrt{2})^2 = 25 \times 2 = 50. \] 因此: \[ AO = \sqrt{54 + 50} = \sqrt{104} = 2\sqrt{26}. \] 由于 \(AO\) 是经过点 \(A\) 、 \(B\) 、 \(C\) 的圆的直径,该圆的半径为: \[ R = \frac{AO}{2} = \frac{2\sqrt{26}}{2} = \sqrt{26}. \]
(A) \(y\) 轴 (B) 直线 \(x = 1\) (C) 原点 (D) 点 \(\left( {\frac{1}{2},0}\right)\) (E) 点 \((1,0)\)
对称性分析的关键是检验函数是否满足特定对称条件。
步骤 1:设 \(x = n + r\) ,其中 \(n\) 为整数, \(0 \leq r < 1\) 将 \(x\) 分解为整数部分 \(n\) 和小数部分 \(r\) ,便于分析。
步骤 2:分情况讨论 \(f(x)\) 的值
步骤 3:检验对称性条件 对称关于点 \(\left( \frac{1}{2}, 0 \right)\) 的条件是: \[ f(1 - x) = -f(x) \] 计算 \(f(1 - x)\) : \[ f(1 - x) = \left| \lfloor 1 - x \rfloor \right| - \left| \lfloor 1 - (1 - x) \rfloor \right| = \left| \lfloor 1 - x \rfloor \right| - \left| \lfloor x \rfloor \right| = -f(x) \] 因此,对任意 \(x\) ,有: \[ f(1 - x) = -f(x) \] 这说明点 \((x, f(x))\) 和点 \((1 - x, -f(x))\) 关于点 \(\left( \frac{1}{2}, 0 \right)\) 对称,因为该点是这两点的中点。
步骤 4:排除其他选项 通过具体值验证其他对称性不成立:
(A) 9 (B) 10 (C) 11 (D) 13 (E) 17
由于平板不平行于地面,支柱顶端共面,因此平板可视为一个平面。设六边形边长为 \(s\) ,建立坐标系,利用共面条件求解。
将正六边形置于平面 \(z=0\) ,中心在原点,顶点按逆时针排列: \[ A = (s, 0, 0), \quad B = \left(\frac{s}{2}, \frac{\sqrt{3}}{2}s, 0\right), \quad C = \left(-\frac{s}{2}, \frac{\sqrt{3}}{2}s, 0\right), \] \[ D = (-s, 0, 0), \quad E = \left(-\frac{s}{2}, -\frac{\sqrt{3}}{2}s, 0\right), \quad F = \left(\frac{s}{2}, -\frac{\sqrt{3}}{2}s, 0\right). \]
已知支柱高度: \(z_A = 12\) , \(z_B = 9\) , \(z_C = 10\) 。
设平板平面方程为: \[ z = px + qy + r. \] 代入 \(A, B, C\) 坐标:
由 (1) 得 \(r = 12 - ps\) 。
(2) 减 (1): \[ 9 - 12 = p\frac{s}{2} - ps + q\frac{\sqrt{3}}{2}s \] \[ -3 = -\frac{p s}{2} + q\frac{\sqrt{3}}{2}s \] \[ -3 = s\left(-\frac{p}{2} + \frac{\sqrt{3}}{2}q\right) \quad (4) \]
(3) 减 (1): \[ 10 - 12 = -p\frac{s}{2} - ps + q\frac{\sqrt{3}}{2}s \] \[ -2 = -\frac{3ps}{2} + q\frac{\sqrt{3}}{2}s \] \[ -2 = s\left(-\frac{3p}{2} + \frac{\sqrt{3}}{2}q\right) \quad (5) \]
(5) 减 (4): \[ -2 - (-3) = s\left[-\frac{3p}{2} + \frac{\sqrt{3}}{2}q + \frac{p}{2} - \frac{\sqrt{3}}{2}q\right] \] \[ 1 = s(-p) \] \[ p = -\frac{1}{s}. \]
代入 (4): \[ -3 = s\left[-\frac{1}{2}\cdot\left(-\frac{1}{s}\right) + \frac{\sqrt{3}}{2}q\right] \] \[ -3 = s\left[\frac{1}{2s} + \frac{\sqrt{3}}{2}q\right] \] \[ -3 = \frac{1}{2} + \frac{\sqrt{3}}{2} s q \] \[ -\frac{7}{2} = \frac{\sqrt{3}}{2} s q \] \[ q = -\frac{7}{\sqrt{3}\, s}. \]
由 (1): \[ r = 12 - \left(-\frac{1}{s}\right) s = 12 + 1 = 13. \]
因此平面方程: \[ z = -\frac{1}{s}x - \frac{7}{\sqrt{3}\, s}y + 13. \]
\(E = \left(-\frac{s}{2}, -\frac{\sqrt{3}}{2}s, 0\right)\) ,代入平面方程: \[ z_E = -\frac{1}{s}\cdot\left(-\frac{s}{2}\right) - \frac{7}{\sqrt{3}\, s} \cdot \left(-\frac{\sqrt{3}}{2}s\right) + 13 \] \[ z_E = \frac{1}{2} + \frac{7}{2} + 13 \] \[ z_E = 4 + 13 = 17. \]
\[ \boxed{17} \]
如图,某农场主将一块矩形田地划分为 \(2 \times 2\) 的网格,共4个矩形区域。每个区域将种植一种作物:玉米、小麦、大豆或马铃薯。农场主不希望任何相邻区域(共享边界)种植玉米和小麦,也不希望任何相邻区域种植大豆和马铃薯。在这些限制条件下,农场主有多少种不同的作物种植方案?
(A) 12 (B) 64 (C) 84 (D) 90 (E) 144
农场主有一块 \(2 \times 2\) 的矩形田地,划分为 4 个区域,按顺时针方向依次编号为 1(左上)、2(右上)、3(右下)、4(左下)。每个区域种植一种作物:玉米(C)、小麦(W)、大豆(S)或马铃薯(P)。限制条件为:
相邻关系为:1 与 2、2 与 3、3 与 4、4 与 1 相邻(对角线不相邻)。
目标是计算满足条件的种植方案总数。
采用分情况讨论法,按区域顺序依次选择作物。
区域 3 与区域 1 不相邻,但区域 2 与区域 1、区域 3 都相邻,区域 4 与区域 1、区域 3 都相邻。因此区域 3 的作物选择会影响区域 2 和 4 的可用选项。
设区域 1 的作物为 \(X\) ,则:
情况 1:区域 3 种植与区域 1 相同的作物 \(X\)
情况 2:区域 3 种植与区域 1 不相容的作物 \(Y\)
情况 3:区域 3 种植 \(Z_1\) 或 \(Z_2\) (两种中的一种)
满足条件的种植方案总数为 \(\boxed{84}\) ,对应选项 (C)。
一个半径为1的圆盘在边长为 \( s > 4 \) 的正方形内部紧贴边界滚动一周,扫过的区域面积为 \( A \) 。另一个半径为1的圆盘在同一个正方形外部紧贴边界滚动一周,扫过的区域面积为 \( 2A \) 。若 \( s \) 可表示为 \( a + \frac{b\pi }{c} \) 的形式,其中 \( a,b,c \) 均为正整数且 \( b \) 与 \( c \) 互质,求 \( a + b + c \) 的值?
(A) 10 (B) 11 (C) 12 (D) 13 (E) 14
内部滚动的圆盘中心轨迹是一个正方形,其边长为 \(s - 2\) (因为圆盘半径 1,中心到边的距离为 1,所以中心轨迹正方形比原正方形边长少 2)。但扫过的区域并不是简单的环形,而是原正方形去掉中心一个边长为 \(s - 4\) 的正方形(因为圆盘贴着内部滚动时,中心离边界 1,圆盘边缘离边界 0,所以中心轨迹内侧距离原边界 2,因此未扫到的中心区域边长是 \(s - 4\) ),并且四个角上未扫到的区域是四个 \(1 \times 1\) 正方形去掉四分之一圆(半径 1)。
更直接的方法(根据答案提示):
因此: \[ \text{未扫到的总面积} = (s - 4)^2 + (4 - \pi) \] \[ A = s^2 - \left[(s - 4)^2 + (4 - \pi)\right] \] \[ A = s^2 - (s^2 - 8s + 16 + 4 - \pi) \] \[ A = s^2 - s^2 + 8s - 20 + \pi \] \[ A = 8s - 20 + \pi \]
外部滚动的圆盘中心轨迹是边长为 \(s + 2\) 的正方形(中心到原正方形边的距离为 1,所以轨迹边长 \(s + 2\) )。扫过的区域是四个矩形(每个长 \(s\) 、宽 2)加上四个四分之一圆(半径 2)。
四个矩形面积: \(4 \times (s \times 2) = 8s\) 四个四分之一圆(半径 2)面积: \(4 \times \frac{1}{4} \pi (2^2) = 4\pi\)
所以: \[ \text{外部滚动扫过的面积} = 8s + 4\pi \] 已知它等于 \(2A\) : \[ 8s + 4\pi = 2(8s - 20 + \pi) \]
\[ 8s + 4\pi = 16s - 40 + 2\pi \] \[ 4\pi - 2\pi + 40 = 16s - 8s \] \[ 2\pi + 40 = 8s \] \[ s = 5 + \frac{\pi}{4} \]
因此 \(a = 5, b = 1, c = 4\) ,且 \(b, c\) 互质。
\[ a + b + c = 5 + 1 + 4 = 10 \]
(A) 4 (B) 6 (C) 8 (D) 12 (E) 16
一个一元二次方程没有两个不同的实数解,当且仅当其判别式小于或等于零。因此,我们需要: \[ b^{2} - 4c \leq 0 \quad \text{和} \quad c^{2} - 4b \leq 0。 \]
由 \(b^{2} \leq 4c\) 和 \(c^{2} \leq 4b\) ,代入 \(c^{2} \leq 4b\) 到第一个不等式: \[ b^{2} \leq 4c \quad \Rightarrow \quad c \geq \frac{b^{2}}{4}。 \] 同时, \(c^{2} \leq 4b\) 意味着 \(c \leq 2\sqrt{b}\) 。因此: \[ \frac{b^{2}}{4} \leq c \leq 2\sqrt{b}。 \] 由于 \(c\) 是正整数,且 \(\frac{b^{2}}{4} \leq 2\sqrt{b}\) ,两边乘以 4: \[ b^{2} \leq 8\sqrt{b} \quad \Rightarrow \quad b^{4} \leq 64b \quad \Rightarrow \quad b^{3} \leq 64。 \] 所以: \[ b \leq 4。 \] 因此, \(b\) 的可能取值为 \(1, 2, 3, 4\) 。
将20个球独立且随机地投入5个箱子中。设 \( p \) 为某个箱子最终有3个球、另一个箱子有5个球、其余三个箱子各有4个球的概率。设 \( q \) 为每个箱子最终都有4个球的概率。求 \( \frac{p}{q} \) 的值。
(A) 8 (B) 10 (C) 12 (D) 14 (E) 16
题目描述将 20 个球独立且随机地投入 5 个箱子中,每个球落入每个箱子的概率均为 \(\frac{1}{5}\) 。
要求计算 \(\frac{p}{q}\) 的值。
由于每个球独立且随机地落入 5 个箱子,总的分配方式数为 \(5^{20}\) ,但概率计算时分子分母都会出现 \(5^{20}\) ,因此 \(\frac{p}{q}\) 实际上等于两种分配方式数的比值。
我们使用多项式系数方法计算两种情况的分配方式数。
每个箱子 4 个球,分配方式数为:
\[ N_q = \frac{20!}{4! \cdot 4! \cdot 4! \cdot 4! \cdot 4!} \]
某个箱子有 3 个球,另一个箱子有 5 个球,其余三个箱子各有 4 个球。
分配方式数为:
\[ N_p = 20 \times \frac{20!}{3! \cdot 5! \cdot 4! \cdot 4! \cdot 4!} \]
由于概率 \(p = \frac{N_p}{5^{20}}\) , \(q = \frac{N_q}{5^{20}}\) ,所以:
\[ \frac{p}{q} = \frac{N_p}{N_q} \]
代入 \(N_p\) 和 \(N_q\) :
\[ \frac{p}{q} = \frac{20 \times \frac{20!}{3! \cdot 5! \cdot (4!)^3}}{\frac{20!}{(4!)^5}} \]
约去 \(20!\) :
\[ \frac{p}{q} = 20 \times \frac{(4!)^5}{3! \cdot 5! \cdot (4!)^3} \]
化简:
\[ \frac{p}{q} = 20 \times \frac{(4!)^2}{3! \cdot 5!} \]
计算阶乘:
\[ \frac{p}{q} = 20 \times \frac{576}{6 \times 120} = 20 \times \frac{576}{720} \]
\[ \frac{576}{720} = \frac{4}{5} \]
\[ \frac{p}{q} = 20 \times \frac{4}{5} = 16 \]
\[ \boxed{16} \]
对应选项 (E)。
问题 在底面半径为5、高为12的直圆锥内部放置三个半径为 \( r \) 的全等球体。每个球体与其他两个球体相切,并且与圆锥的底面和侧面相切。求 \( r \) 的值。
(A) \( \frac{3}{2} \) (B) \( \frac{90 - 40\sqrt{3}}{11} \) (C) 2 (D) \( \frac{144 - 25\sqrt{3}}{44} \) (E) \( \frac{5}{2} \)